APMO Power!

If a a and b b are positive integers from 1 1 to 999 999 [inclusive], how many ordered pairs ( a , b ) (a, b) are there such that ( a + 36 b ) ( b + 36 a ) (a+36b)(b+36a) is an integral power of 2 2 ?


This problem is a reworded version of a problem that appeared at the APMO.


This problem is a part of the set "Olympiads and Contests Around the World - 2". You can see rest of the problems here .


The answer is 0.

This section requires Javascript.
You are seeing this because something didn't load right. We suggest you, (a) try refreshing the page, (b) enabling javascript if it is disabled on your browser and, finally, (c) loading the non-javascript version of this page . We're sorry about the hassle.

7 solutions

Mathh Mathh
Jul 12, 2014

In this proof the set of positive integers will be called N \mathbb N . This proof uses a popular method called Proof by Infinite Descent .

Proof by contradiction: Say it is possible that ( a + 36 b ) ( b + 36 a ) = 2 x \displaystyle (a+36b)(b+36a)=2^{x} , where a , b , x N a,b,x\in\mathbb N .

Notice that a + 36 b = 2 n a+36b=2^n and b + 36 a = 2 m b+36a=2^m , where n , m N n,m\in\mathbb N .

Hence a , b a,b are even. Let a = 2 a , b = 2 b a=2a', b=2b' , where a , b N a',b'\in\mathbb N .

( 2 a + 36 ( 2 b ) ) ( 2 b + 36 ( 2 a ) ) = 4 ( a + 36 b ) ( b + 36 a ) = 2 x \displaystyle (2a'+36(2b'))(2b'+36(2a'))=4(a'+36b')(b'+36a')=2^{x}

( a + 36 b ) ( b + 36 a ) = 2 x 2 \displaystyle\implies (a'+36b')(b'+36a')=2^{x-2}

Notice that ( a + 36 b ) ( b + 36 a ) N \displaystyle (a'+36b')(b'+36a')\in\mathbb N . Hence x 2 = x N x-2=x'\in\mathbb N .

Therefore, we have ( a + 36 b ) ( b + 36 a ) = 2 x \displaystyle (a'+36b')(b'+36a')=2^{x'} , where a , b , x N a',b',x'\in\mathbb N .

But look at the beginning of the proof... this is the same problem we started with! Except the variables now carry smaller values. But they can't decrease forever, because the positive integers have a minimum value, namely 1 1 , which will be reached at some point.

Therefore, we have a contradiction - there are no such variables a , b , x N a,b,x\in\mathbb N satisfying ( a + 36 b ) ( b + 36 a ) = 2 x (a+36b)(b+36a)=2^x . \square

Method of Infinite Descent. I think this is what @Mursalin Habib was looking for.

Daniel Liu - 6 years, 11 months ago

Log in to reply

Yes, this is it. I do have another solution though...

Mursalin Habib - 6 years, 11 months ago

Log in to reply

Does the other solution involve taking out all powers of $2$ from $a$ and $b$ and then derive a contradiction?

Rahul Saha - 6 years, 10 months ago

We'd be glad to hear any hints about your solution, it might teach us all something new.

mathh mathh - 6 years, 10 months ago
Anatoliy Razin
Nov 19, 2014

a + 36 b a + 36b and b + 36 a b + 36a are powers of 2 37 ( a + b ) = 2 m ( 2 n + 1 ) 2 \Rightarrow 37(a + b) = 2^m(2^n + 1) for some m , n m, n

therefore 2 n 1 ( m o d 37 ) n 18 2^n \equiv -1 \pmod{37} \Rightarrow n \geq 18 , however 37 ( a + b ) < 2 17 37(a+b) < 2^{17}

Avijit Sarker
Jul 15, 2014

I used the idea that, if 2 m = a + 36 b 2^{m} = a + 36b and 2 n = b + 36 a 2^{n} = b + 36a , and m > = n m >= n , then, 2 n ( 2 m n 1 ) = 35 ( b a ) 2^{n} (2^{m-n} - 1) = 35 (b - a) Hence, we can say, m n = 12 d m - n = 12d , because the order of 2 mod 35 is 12. Continuing, ( b a ) = 2 n ( 2 12 d 1 ) 35 (b - a) = \frac{2^{n} (2^{12d} - 1)}{35} As, a , b < = 999 a, b <= 999 , we can say, d = 1 d = 1 Now, b a = 2 n 117 b - a = 2^{n} 117 And, b + 36 a = 2 n b + 36a = 2^{n} Using these two equations, we find, 37 a = 2 n ( 116 ) 37a = 2^{n}(-116) Which is a contradiction(a is an integer). Hence, no solution.

a a being an integer does not contradict anything, a a being positive does. But a great solution otherwise. +1

mathh mathh - 6 years, 11 months ago
Mehul Gajwani
Jul 12, 2014

We can see that a + 36 b a + 36b and b + 36 a b + 36a must both be powers of two. Hence a + 36 b = 2 n a + 36b = 2^n and b + 36 a = 2 m b + 36a = 2^m .

Adding yields 37 ( a + b ) = 2 n + 2 m a + b = 2 n + 2 m 37 37(a+b) = 2^n + 2^m \rightarrow a + b = \frac{2^n + 2^m}{37} . The numerator will always be even and the denominator will always be odd, so there will be no integer solutions to a and b.

In addition to Daniel's comment, there is no issue with the numerator being even and the denominator being odd. For example, 6 3 \frac{6}{3} is an integer.

The best you can hope for, is that the numerator is not a multiple of 37. However, this is not true, since 2 19 + 2 1 2^{19} + 2^1 is a multiple of 37.

Calvin Lin Staff - 6 years, 11 months ago

The numerator dos not necessarily have to be even. We can have either m m or n n equal 0 0 and the numerator will be odd.

Daniel Liu - 6 years, 11 months ago

nice solution Mr. Mahul

Rajeev Giri - 6 years, 11 months ago
Kenny Lau
Jul 21, 2015

I will skip some obvious steps, since this is a level 4 question.

  • a + 36 b = 2 x a+36b=2^x --1
  • b + 36 a = 2 y b+36a=2^y --2
  • 1&2: a = 36 × 2 y 2 x 3 6 2 1 a=\dfrac{36\times2^y-2^x}{36^2-1} --3
  • 1&2: b = 36 × 2 x 2 y 3 6 2 1 b=\dfrac{36\times2^x-2^y}{36^2-1} --4

We will look for x x and y y such that a a and b b are (positive) integers.

  • 3: ( 3 6 2 1 ) ( 36 × 2 y 2 x ) (36^2-1)|(36\times2^y-2^x)
  • 1295 ( 36 × 2 y 2 x ) 1295|(36\times2^y-2^x)

Prime factors of 1295 1295 are 5 5 , 7 7 , 37 37 .

  • 5 ( 36 × 2 y 2 x ) \implies 5|(36\times2^y-2^x) --5
  • 7 ( 36 × 2 y 2 x ) \implies 7|(36\times2^y-2^x) --6
  • 37 ( 36 × 2 y 2 x ) \implies 37|(36\times2^y-2^x) --7

Then:

  • 5: 5 ( 2 y 2 x ) \implies 5|(2^y-2^x) --8
  • 6: 7 ( 2 y 2 x ) \implies 7|(2^y-2^x) --9
  • 7: 37 ( 2 y + 2 x ) \implies 37|(2^y+2^x) --10

  • Observe (powers of 2) mod 5: 1 , 2 , 4 , 3 , 1 , 1,2,4,3,1,\cdots

  • Observe (powers of 2) mod 7: 1 , 2 , 4 , 1 , 1,2,4,1,\cdots
  • Observe (powers of 2) mod 37:
  • 1 , 2 , 4 , 8 , 16 , 32 , 27 , 17 , 34 , 31 , 25 , 13 , 26 , 15 , 30 , 23 , 9 , 18 , 1 , 2 , 4 , 8 , 16 , 32 , 27 , 17 , 34 , 31 , 25 , 13 , 26 , 15 , 30 , 23 , 9 , 18 , 1,2,4,8,16,32,27,17,34,31,25,13,26,15,30,23,9,18,-1,-2,-4,-8,-16,-32,-27,-17,-34,-31,-25,-13,-26,-15,-30,-23,-9,-18,\cdots

Therefore:

  • 8: y x = 4 p y-x=4p --11
  • 9: y x = 3 q y-x=3q --12
  • 10: y x = 18 + 36 r y-x=18+36r --13

Thus:

  • 11&12&13: y x = 12 m = 18 + 36 r y-x=12m=18+36r
  • 2 m = 3 + 6 r \implies 2m=3+6r
  • even = odd + even \implies \mbox{even}=\mbox{odd}+\mbox{even}
  • \implies no solution
Jakub Šafin
Jan 27, 2015

Let a + 36 b = 2 k , b + 36 a = 2 l a+36b=2^k, b+36a=2^l . Then, 37 ( a + b ) = 2 l + 2 k = 2 k ( 2 l k + 1 ) 37(a+b)=2^l+2^k=2^k(2^{l-k}+1) w.l.o.g.

For this to hold, we need 37 37 to divide 2 l k + 1 2^{l-k}+1 . But we also need 2 l < 37 999 2^l < 37\cdot999 , so 0 l k l 16 0 \le l-k \le l \le 16 . Computing the first few powers of 2 l k + 1 2^{l-k}+1 modulo 37, we can see that none give remainder 0, so there can't be any solutions.

Sunil Bn
Aug 20, 2014

JavaScript for(var i =1;i<1000;i++){ for(var j =1;j<1000;j++){ var val = (i+36 j) (j+36*i); var temp = val.toString(2); if( temp % 2 == 0){ var count = temp.match(/1/g); if(count == 1) console.log(temp); } } }

0 pending reports

×

Problem Loading...

Note Loading...

Set Loading...